truongxuan2016-2

Embed Size (px)

Citation preview

  • 8/17/2019 truongxuan2016-2

    1/18

    Trưng Xuân Toán hc min Nam 2016

    VIT NAM TST MOCK TEST

    Thi gian làm bài: 240 phút.

    1 Đ thi ngày 1 (ngày 24/02/2016)

    Bài 1.

    Cho  a,b, c là các s thc dương. Chng minh rng ta có bt đng thc

    ab + bc + ca +√ 

    a2b2 + b2c2 + c2a2

    a + b + c + √ a2 + b2 + c2   ≤  a + b + c

    3Bài 2.

    Cho tam giác  ABC  nhn ni tip trong  (O), ngoi tip (I ). Gi (J ) là đưng tròn

    Euler và   H   là trc tâm tam giác. Đưng tròn   (I ) tip xúc vi  AB, AC  ln lưt

    ti  D, E . Đim  T  di đng trên  (J ) và đưng thng qua  T  vuông góc vi  HT   ct

    (O)   M, N . Dng hình bình hành  MH NK  .

    1. Chng minh rng  K  luôn di chuyn trên mt đưng c đnh khi  T  thay đi.

    2. Đưng tròn (S ) tip xúc ngoài vi  (J ) và tip xúc vi các đon thng  AB,AC 

    ln lưt ti   X, Y . Gi   Z  là trc tâm ca tam giác   ADE . Chng minh rng

    t giác   AXZY  là hình thoi.

    Bài 3.

    Mt cp s cng các s nguyên dương gm ít nht  3 s hng đưc gi là  chun

    nu tích các s hng ca nó là ưc s ca mt s có dng  n2 + 1.

    1. Chng minh rng tn ti mt cp s cng chun vi công sai  12.2. Chng minh rng không tn ti cp s cng chun vi công sai  10 và  11.

    3. Hi mt cp s cng chun vi công sai bng  12 có th có nhiu nht bao

    nhiêu s hng?

  • 8/17/2019 truongxuan2016-2

    2/18

    2 Đ thi ngày 2 (ngày 26/02/2016)

    Bài 4.

    1. Cho bng hình ch nht  m × n ô vi  m, n là các s nguyên dương cho trưc.Trên mi ô ca bng ta vit  1 trong các s  0, 1, 2 sao cho tng các s trên

    mi hàng, mi ct chia ht cho  3. Hi có th có nhiu nht bao nhiêu s  1?

    2. Cho hình hp ch nht  2015 × 2016 × 2017 đưc to thành t các hình lpphương đơn v. Trong mi hình lp phương đơn v, ta vit mt trong các

    s  0, 1, 2 sao cho tng các s trong mi dài  1 × 1 × 1 × 2017, 1 × 2016 × 1 và2015 × 1 × 1 chia ht cho  3. Hi có th có nhiu nht bao nhiêu s  1?

    Bài 5.Cho tam giác   ABC  ni tip đưng tròn   (O) bán kính   R, ngoi tip đưng tròn

    (I ) bán kính  r  và có các đưng trung tuyn là  AA1, BB1, CC 1. Tip tuyn ti B , C 

    ca đưng tròn  (O) ct nhau ti  S  và gi s  AS  ct  BC  ti  A2. Các đim  B2, C 2đưc xác đnh tương t. Chng minh rng

    AA2

    AA1+

     BB2

    BB1+

     CC 2

    CC 1≥ 1 +  4r

    R.

    Bài 6.Vi s nguyên dương  n, gi  Bn+1 là tp tt c các xâu nh phân đ dài  n, tc là

    Bn+1 = {anan−1 . . . a0 | ai ∈ {0, 1}, ∀i = 0, 1, . . . , n} .

    Vi mi xâu  a =  anan−1 . . . a0 thuc  Bn+1 ta gi

    s(a) = an + an−1 + · · · + a0   (mod 2) là bit kim tra ca xâu  a

    v(a) = an2n + an−12

    n−1 + · · · + a1 · 2 + a0 là giá tr ca xâu  a

    Gi   Bn+1

    0   , Bn+1

    1   tương ng là tp hp tt c các xâu nh phân có đ dài   n + 1có bit kim tra tương ng là  0 và  1. Chng minh rng vi mi s nguyên dương

    k, n, ta có đng thc

    a∈Bn+10

    (v(a))k =

    a∈Bn+11

    (v(a))k.

  • 8/17/2019 truongxuan2016-2

    3/18

    ĐÁP ÁN CHI TIT

    Bài 1.  Cho   a,b,c  là các s thc dương. Chng minh rng ta có bt đng thc 

    ab + bc + ca +√ 

    a2b2 + b2c2 + c2a2

    a + b + c +√ 

    a2 + b2 + c2  ≤   a + b + c

    3

    Li gii.  Ta cn chng minh

    ab + bc + ca +√ 

    a2b2 + b2c2 + c2a2

    a + b + c +√ 

    a2 + b2 + c2  ≤   a + b + c

    3

    Khai trin và rút gn, ta có

    a2 + b2 + c2 − (a + b + c) a2 + b2 + c2 ≤ a2 + b2 + c2 − ab − bc − ca  (∗).Ta xét 2 trưng hp:

    1. Nu  3√ 

    a2 + b2 + c2 ≤ (a + b + c)√ 

    a2 + b2 + c2 thì chú ý rng

    a2 + b2 + c2 − ab − bc − ca =  12

    (a − b)2 + (b − c)2 + (c − a)2

    nên d thy  V P (∗) ≥ 0 ≥ V T (∗). Do đó bt đng thc  (∗) đúng.

    2. Nu  3√ a2 + b2 + c2 > (a + b + c)√ a2 + b2 + c2, t  (∗), ta cn chng minh:9(a2b2 + b2c2 + c2a2) − (a + b + c)2(a2 + b2 + c2)

    2(a + b + c)√ 

    a2 + b2 + c2  ≤ a2 + b2 + c2 − ab − bc − ca

    ⇔−

    cyc(a − b)2

    a2 + b2 + 6ab − 2c2

    2(a + b + c)

    √ a2 + b2 + c2

      ≤cyc

    (a − b)2

    Ta cn có

    −cyc

    (a − b)2

    a2

    + b2

    + 6ab − 2c2 ≤ 2(a + b + c) a2 + b2 + c2

    cyc(a − b)

    2

    (∗∗)

    Không mt tính tng quát, ta gi s  a = max {a,b,c}.

    D thy

    2(a + b + c) 

    a2 + b2 + c2 > 2a2

  • 8/17/2019 truongxuan2016-2

    4/18

    nên  V P (∗∗)  > 2a2cyc

    (a − b)2 ≥ 2a2(b − c)2 + 2b2(c − a)2 + 2c2(a − b)2 = T .

    Chú ý rng

    V T (∗∗) − T   = −cyc

    (a − b)2 a2 + b2 + 6ab ≤ 0nên suy ra

    V T (∗∗) ≤ T < V P  (∗∗).

    Do đó, (**) đúng.

    Bt đng thc đã cho đưc chng minh.

    Đng thc xy ra khi  a =  b  =  c.  

    Bài 2.  Cho tam giác   ABC   nhn ni tip trong   (O), ngoi tip   (I ). Gi   (J )   là 

    đưng tròn Euler và  H  là trc tâm tam giác. Đưng tròn  (I ) tip xúc vi  AB,AC 

    ln lưt ti   D, E . Đim   T  di đng trên   (J )  và đưng thng qua   T  vuông góc vi 

    HT   ct   (O)     M, N . Dng hình bình hành   MH NK  .

    1. Chng minh rng  K   luôn di chuyn trên mt đưng c đnh khi  T  thay đi.

    2. Đưng tròn  (S ) tip xúc ngoài vi  (J ) và tip xúc vi các đon thng  AB,AC 

    ln lưt ti   X, Y . Gi   Z   là trc tâm ca tam giác   ADE . Chng minh rng 

    t giác   AXZY   là hình thoi.

    Li gii.  1) Ta bit rng

    HJ 

    HO  =

     1

    2 và

      R(J )

    R(O)=

     1

    2

    nên phép v t tâm  H, t s   12  bin  (O) thành  (J ).

    Gi  R là trung đim  M N  thì  R cũng là trung đim  HK . Do  O là tâm ca đưngtròn nên  OR ⊥ M N  và t giác  HORT  là hình thang vuông.

    Chú ý rng  J  là trung đim ca  HO nên d dàng chng minh đưc

    JT   = J R hay  R ∈ (J ).

  • 8/17/2019 truongxuan2016-2

    5/18

    Phép v t đã nêu bin  K  thành  R, mà  R ∈ (J ) nên  K  ∈ (O).Do đó,  K  luôn thuc đưng tròn  (O) c đnh.

    2) Trưc ht, ta s chng minh b đ sau:

    B đ. Cho tam giác  ABC  có  D, E  ln lưt là tip đim ca đưng tròn ni tip(I ) lên  AB, AC  và  H, K   ln lưt là hình chiu ca   B   lên   AC  và  C   lên   AB.  Khi

    đó, tâm đưng tròn ni tip ca  ∆AHK  là trc tâm ca  ∆ADE.

    Chng minh.

  • 8/17/2019 truongxuan2016-2

    6/18

    Gi J, J  ln lưt là tâm đưng tròn ni tip  ∆AHK  và trc tâm ∆ADE. D thy

    rng  ∆AHK  ∼ ∆ACB theo t l   HK BC  = cos A nên  AJ  = AI  cos A.

    Hơn na, ta thy rng   ∆ADE  ni tip đưng tròn đưng kính AI nên   AI  =

    AI  cos A. Do   J, J  cùng thuc phân giác góc   A  và   AJ   =  AJ  nên   J  ≡  J . B đđưc chng minh.

    Tr li bài toán,

    Gi  B, C  ln lưt là chân đưng cao đnh  B, C  ca tam giác  ABC  và B, C  ln

    lưt là trung đim ca  AC, AB. Rõ ràng  B, C , B, C  ∈ (J ).

    Xét phép nghch đo  Ω tâm  A, phương tích là  k = P A/(J ). Khi đó:

    Ω : (J ) → (J ).Gi  P, Q ln lưt là trung đim ca  AC , AB thì d thy rng

    k =  AC  · AC  = 2AP  ·  12

    AB = AP  · AB

    nên  Ω :  B → P .

    Tương t,  Ω : C  → Q nên  Ω :  B C  → (AP Q).

  • 8/17/2019 truongxuan2016-2

    7/18

    Gi (L) là đưng tròn bàng tip góc A ca tam giác ABC  thì d thy Ω : (L) → (S )là đưng tròn A-Mixilinear ca tam giác   APQ.

    Theo đnh lý Feuerbach thì (L) tip xúc vi  (J ) nên  (S ) tip xúc vi  (J ), mà  (S )

    cũng tip xúc vi  AB,AC  nên  (S ) ≡ (S ).Gi  F  là tâm đưng tròn ni tip ca tam giác   ABC  thì theo b đ Sawayama,

    ta có   F   là trung đim   XY.  Áp dng b đ trên, ta thy   Z  cũng là tâm đưng

    tròn ni tip tam giác  ABC .

    Theo tính cht ca phép v t tâm  A, t s   12 bin  ∆ABC  → ∆AQP  thì   AF AZ   =   12 ,

    mà  A, F,Z  cùng thuc phân giác góc  A nên  F  là trung đim  AZ. Do đó, t giác

    AXZY  là hình bình hành.

    D thy  AX  = AY   nên  AXZY  là hình thoi. Ta có đpcm.  

    Bài 3.  Mt cp s cng các s nguyên dương gm ít nht  3  s hng đưc gi là 

    chun nu tích các s hng ca nó là ưc s ca mt s có dng  n2 + 1.

    1. Chng minh rng tn ti mt cp s cng chun vi công sai  12.

    2. Chng minh rng không tn ti cp s cng chun vi công sai  10  và  11.

    3. Hi mt cp s cng chun vi công sai bng   12  có th có nhiu nht bao

    nhiêu s hng? 

    Li gii.  1) Ta chn cp s cng (CSC)   1, 13, 25 có công sai  12 và đ dài là   3.Rõ ràng đây là CSC chun.

    D thy  1 · 13 · 25 = 325 = 182 + 1 nên nó cũng chính là ưc ca s có dng  n2 + 1,tha mãn điu kin đã cho.

    2) Ta có b đ quen thuc sau:

    B đ. S nguyên dương  n2 + 1 không có ưc nguyên t dng  4k + 3  vi  k là snguyên dương.

    •  Chng minh không tn ti CSC chun có công sai là  10.

  • 8/17/2019 truongxuan2016-2

    8/18

    Gi s tn ti CSC như th có s hng đu là  a. Xét  3  s hng liên tip ca

    CSC này bt đu t  a là  a, a + 10, a + 20, d thy rng

    a(a + 10)(a + 20)

     ≡ a(a + 1)(a + 2)

     ≡ 0 (mod 3)

    vì  a(a +1)(a + 2) là tích ca  3  s nguyên dương liên tip. Suy ra tích ca các

    s hng này chia ht cho 3, tc là có ưc nguyên t dng  4k + 3, theo b đ

    trên thì nó không tha mãn.

    •  Chng minh không tn ti CSC chun có công sai là  11.

    Ta thy

    a(a + 11)(a + 22) ≡ a(a + 2)(a + 1) ≡ 0 (mod 3)

    nên trưng hp này tương t.

    Do đó, không tn ti CSC chun có công sai là  10 hay  11.

    3) Vi   x là s hng đu ca CSC công sai là  12, ta xét  7 s hng đu ca CSC

    này là:

    x, x + 12, x + 24, x + 36, x + 48, x + 60, x + 72.

    Rõ ràng   (12, 7) = 1  nên các s trên lp thành mt h thng dư đy đ theo

    modulo  7, nghĩa là tích ca chúng s chia ht cho  7.Tuy nhiên  7 là s nguyên t có dng  4k + 3 nên cũng không tn ti s  n sao cho

    n2 + 1 chia ht cho tích các s hng này.

    Do đó, CSC chun có công sai là  12 phi có s các s hng không vưt quá  6.

    Tip theo, ta li xét b s  (5, 17, 29, 41, 53, 65). Kim tra trc tip, ta thy:

    n ≡ 7 (mod 25) ⇒ 25n2 + 1

    n ≡ 5 (mod 13) ⇒ 13 n2 + 1n ≡ 12 (mod 29) ⇒ 29

    n2 + 1n ≡ 9 (mod 41) ⇒ 41

    n2 + 1n ≡ 23 (mod 53) ⇒ 53

    n2 + 1

  • 8/17/2019 truongxuan2016-2

    9/18

    Do đó, xét s  n tha mãn h

    n ≡ 7 (mod 25)n

     ≡ 5 (mod 13)

    n ≡ 12 (mod 29)n ≡ 9 (mod 41)n ≡ 23 (mod 53)

    thì theo đnh lý thng dư Trung Hoa (vì các modulo đôi mt nguyên t cùng

    nhau), ta thy

    n2 + 1 ≡ 0 (mod 25 · 13 · 17 · 29 · 41 · 53)

    hayn2 + 1 ≡ 0 (mod 5 · 17 · 29 · 41 · 53 · 65).

    Suy ra CSC  5, 17, 29, 41, 53, 65 là mt CSC chun có đ dài là  6.

    Vy 6 là đ dài ln nht cn tìm.  

    Bài 4.   1. Cho bng   m × n  ô vi   m, n   là các s nguyên dương cho trưc. Trên mi ô ca bng ta vit   1   trong các s   0, 1, 2   sao cho tng các s trên mi 

    hàng, mi ct chia ht cho  3. Hi có th có nhiu nht bao nhiêu s  1? 

    2. Cho hình hp ch nht   2015 × 2016 × 2017  đưc to thành t các hình lpphương đơn v. Trong mi hình lp phương đơn v, ta vit mt trong các 

    s  0, 1, 2  sao cho tng các s trong mi dài   1 × 1 × 1 × 2017, 1 × 2016 × 1  và 2015 × 1 × 1 chia ht cho  3. Hi có th có nhiu nht bao nhiêu s  1? 

    Li gii.  Gi  f (m, n) là s ln nht các s  1 có th đin vào bng  m × n.Ta xét các trưng hp sau:

    1. Nu  m, n đu chia ht cho  3.Ta đin toàn b bng bi  mn s  1 thì rõ ràng tng các s mi hàng và mi

    ct đu chia ht cho 3, tha mãn đ bài. Rõ ràng  mn cũng là giá tr ln nht

    các s 1 có th đin đưc.

    Suy ra  f (m, n) = mn.

  • 8/17/2019 truongxuan2016-2

    10/18

    2. Nu  m chia ht cho  3, còn  n không chia ht cho  3.

    Ta thy mi hàng có s ô là   n, không chia ht cho   3 nên s các s   1 đin

    đưc nhiu nht là  n−

    1. Do đó, tng s các s  1 đin đưc nhiu nht ca

    bng là  m(n − 1) = mn − m. Đ xây dng đưc bng như th, ta có th thchin như sau:

    •  Nu  n chia  3 dư  1 thì ta đin s theo dng  1 − 1 − 1 − . . . − 1 − 0 (ô cuicùng là  2, còn li đu là  0).

    •  Nu  n chia  3 dư  2 thì ta đin s theo dng  1 − 1 − 1 − . . . − 1 − 2 (ô cuicùng là  2, còn li đu là  1).

    Khi đó, tng các s mi hàng đu chia ht cho  3, còn tng các s  n − 1ct đu đu là  m, tng các s ct cui là  0 hoc  2m, cũng chia ht cho  3.

    Suy ra  f (m, n) = mn − m.

    3. Nu   m  không chia ht cho   3,   còn   n   chia ht cho   3.  Tương t trên, ta có

    f (m, n) = mn − n.

    4. Nu  m, n đu không chia ht cho  3 nhưng chúng có cùng s dư.

    Lp lun tương t trên, ta thy nu xét theo hàng thì mi hàng có khôngquá   n − 1 s  1 nên c bng có không quá   m(n − 1) =  mn − m s  1. Li xéttheo ct thì tng s các s  1  ca bng s không quá  (m − 1)n =  mn − n s  1.

    Suy ra   f (m, n) ≤   mn − max {m, n} .  Ta s ch ra mt cách đin tha mãn.Không mt tính tng quát, gi s  max {m, n} = m.

    Ban đu, ta đin tt c các ô bi s  1. Sau đó, xóa các s  (1, 1), (2, 2), . . . , (n, n)

    và các ô  (n + 1, n), (n + 2, n), . . . , (m, n) ri đin vào đó s  a.

    D dàng thy rng:

    •  Tng các s trên mi hàng là  n − 1 + a.•  Tng các s trên  n − 1 ct đu tiên là  m − 1 + a.•  Tng các s trên ct cui là  m + (a − 1)n.

  • 8/17/2019 truongxuan2016-2

    11/18

    Khi đó, nu   m ≡   n ≡   1(mod3)  thì chn   a   = 0, nu   m ≡   n ≡   2(mod3)   thìchn  a = 2 và bng khi đó tha mãn điu kin đ bài. Dưi đây là ví d cho

    trưng hp  7 × 4 và  8 × 5:

    Do đó,  f (m, n) = mn − max {m, n}.

    5. Trưng hp   m, n  đu không chia ht cho   3  và khác s dư khi chia cho   3.

    Không mt tính tng quát, ta gi s  m ≡ 1 (mod 3) và  n ≡ 2 (mod 3).

    Gi   x, y  ln lưt là s lưng các s  0 và 2 có th đin vào bng. Khi đó, s

    các s  1 đưc đin cho bng là  mn − (x + y).

    Ta thy rng hàng th   i vi  1 ≤ i ≤ m, nu không đin s 0 thì phi đinít nht   1  s   2,  ngưc li, nu có đin s 0 thì phi có ít nht   2  s   0.  Gixi, yi tương ng là s các s  0 và  2 đin cho hàng   i thì t nhn xét trên, ta

    có   xi2   + yi ≥ 1. Suy ra 1≤i≤m

    xi

    2  + yi

    ≥ m ⇒   x

    2 + y ≥ m.

    Lp lun tương t, ta cũng có

    x + y

    2 ≥  n.Do đó  m + n ≤   32(x + y) ⇒ x + y ≥   23(m + n).

    Suy ra s các s  1 đưc đin vào bng s không vưt quá

    mn −  23

    (m + n).

  • 8/17/2019 truongxuan2016-2

    12/18

    Hơn na, cũng theo lp lun trưng hp trên, nu xét theo tng hàng thì

    s các s  1 s không vưt quá   n − 1 và s các s  1 ca bng s không quámn − m. Tương t, nu tính theo tng ct ri cng li thì s các s  1 ca

    bng s không vưt quá  mn − n. Suy raf (m, n) ≤ mn − max

    m,n,

     2

    3(m + n)

    .

    Ta s xây dng bng ô vuông tha mãn đng thc xy ra. Tht vy, ta xét

    các trưng hp c th:

    (a) Nu   max

    m, n, 23(m + n)

     =   n  thì   n >   23(m +  n) ⇔   n >  2m: đin các ô(i, 2i − 1) và  (i, 2i) vi  1 ≤ i ≤ m  s 0, các ô t  (m, 2m + 1) → (m, n) đin

    s   a. D thy tng ca tt c các ct đu là   m − 1 ≡  0(mod3)  (vì mict đin đúng 1 s 0, còn li đu là 1), trong khi đó:•  Tng các s ca  m − 1 hàng đu là  n − 2 ≡ 0(mod3).•  Tng các s ca hàng cui là  2m − 2 ≡ 0(mod3).

    Dưi đây là minh ha cho trưng hp  m = 4, n = 11:

    (b) Nu  max

    m, n, 23(m + n)

    = m  thì đin tương t.

    (c) Nu  max

    m,n,  23(m + n)

     =   23(m + n) thì đt   m = 3r + 1, n  = 3s + 2  vi

    r, s là các s t nhiên. Đng thc phi xy ra trong các đánh giá ca  x, y

    nên  x +   y2  = 3r + 1  và  x2  + y = 3s + 2. Ta tính đưc

    x = 2(2r−

    s + 1), y = 2(2s−

    r).

    Khi đó, trong mi ct ca  2s − r ct đu, ta đin tng cp s  2 vào  2 ôca ct sao cho không có s nào chung hàng thì tng các s ca mi ct

    trong  2s − r ct đó là  m + 2 ≡ 0(mod3).

  • 8/17/2019 truongxuan2016-2

    13/18

    Tương t trên, ta li đin theo các hàng, mi hàng đin  2 s  0 sao cho

    không có s nào chung ct. Khi đó, bng đin đưc s tha mãn tt c

    các yêu cu đt ra. Dưi đây là minh ha cho trưng hp  m = 4, n = 5:

    Do đó, ta có  f (m, n) = mn − max

    m,n,  23(m + n)

    .

    Vy ta có kt lun:

    •  Nu  m ≡ n ≡ 0 (mod 3) thì  f (m, n) = mn.•  Nu  m ≡ 0 (mod 3), n = 0 (mod 3) thì  f (m, n) = mn − m.

    •  Nu  m = 0 (mod 3), n ≡ 0 (mod 3) thì  f (m, n) = mn − n.

    •  Nu  mn = 0 (mod 3), m ≡ n   (mod 3) thì  f (m, n) = mn − max {m, n} .

    •  Nu mn = 0 (mod 3), m = n   (mod 3) thì f (m, n) = mn−max

    m,n,  23(m + n)

    .

    2) Ta chia hình hp thành các di  2015 × 1 × 2017 thì rõ ràng chúng đu là cáchình ch nht có kích thưc  2015 × 2017.Theo câu a, s các ô đin s   1   thuc mi di như th s không vưt quá

    f (2015, 2017). Suy ra tng s các s 1 ca bng s không vưt quá 2016f (2015, 2017).

    Chú ý rng  2016 chia ht cho  3 nên ta ch cn đin s cho  1  di ri nhân bn nó

    lên và ghép li thì s đưc hình hp ch nht tha mãn đ bài.

    Vì  2015, 2017 chia  3 có các s dư khác nhau nên

    f (2015, 2017) = 2015 · 2017 −  23

    (2015 + 2017) = 2015 · 2017 − 2688.

    Vy s các s  1 nhiu nht đin đưc vào hình hp là

    2016(2016 · 2017 − 2688).

  • 8/17/2019 truongxuan2016-2

    14/18

    Bài 5.  Cho tam giác  ABC  ni tip đưng tròn  (O) bán kính  R, ngoi tip đưng 

    tròn  (I ) bán kính  r và có các đưng trung tuyn là  AA1, BB1, CC 1. Tip tuyn ti 

    B, C  ca đưng tròn   (O)  ct nhau ti   S   và gi s   AS   ct   BC   ti   A2. Các đim 

    B2, C 2  đưc xác đnh tương t. Chng minh rng AA2

    AA1+

     BB2

    BB1+

     CC 2

    CC 1≥ 1 +  4r

    R.

    Li gii.  Đt   AB   =  c, BC   =  a, CA  =   b khi đó d thy rng   AA2   là đưng đitrung ca tam giác  ABC, cho nên

    A2B

    A2C   =

      AB2

    AC 2  =

      c2

    b2,

    A2B =  ac2

    b2 + c2 , A2C  =

      ab2

    b2 + c2 .

    Theo h thc Stewart thì

    AA22 = AB2 · A2C  + AC 2 · A2B

    BC   − A2B · A2C 

    =  b2c2

    b2 + c2 +

      b2c2

    b2 + c2 −   a

    2b2c2

    (b2 + c2)2

      =  b2c2(2b2 + 2c2 − a2)

    (b2 + c2)2

      .

    .

  • 8/17/2019 truongxuan2016-2

    15/18

    Mt khác, theo công thc đưng trung tuyn thì  AA21 =  2(b2+c2)−a2

    4   , do đó

    AA22AA21

    = b2c2(2b2 + 2c2 − a2)

    (b2 + c2)2

      : 2b2 + 2c2 − a2

    4  =

      4b2c2

    (b2 + c2)2

    ,

    hay  AA2

    AA1=

      2bc

    b2 + c2.

    Bin đi tương t cho  BB2

    BB1và

      CC 2

    CC 1ta đưa bài toán v chng minh

    bc

    b2 + c2 +

      ca

    c2 + a2 +

      ab

    a2 + b2 ≥  1

    2 +

     2r

    R.

    Vit bt đng thc li như sau

    ab

    a2 + b2  +

      bc

    b2 + c2  +

      ca

    c2 + a2 

     1

    2 +

     (a + b

    −c)(b + c

    −a)(c + a

    −b)

    abc   ,

    hoc

    (a + b)2

    a2 + b2  +

     (b + c)2

    b2 + c2  +

     (c + a)2

    c2 + a2  4 +

     2(a + b − c)(b + c − a)(c + a − b)abc

      .

    Áp dng bt đng thc Cauchy-Schwarz, ta có

    (a + b)2

    a2 + b2  +

     (b + c)2

    b2 + c2  +

     (c + a)2

    c2 + a2 

     2(a + b + c)2

    a2 + b2 + c2  .

    Ta cn ch ra(a + b + c)2

    a2 + b2 + c2  2 +

     (a + b − c)(b + c − a)(c + a − b)abc

      ,

    tương đương vi

    (a + b + c)2  (a2 + b2 + c2)[2abc + (a + b − c)(b + c − a)(c + a − b)]

    abc  .

    Gi s  b là s nm gia  a và  c. Theo bt đng thc AM-GM, ta thy v phi s

    không vưt quá

    1

    4

    a2 + b2 + c2

    b  +

     2abc + (a + b − c)(b + c − a)(c + a − b)ca

    2.

    Như vy ta cn chng minh

    2(a + b + c)  a2 + b2 + c2

    b  +

     2abc + (a + b − c)(b + c − a)(c + a − b)ca

      ,

  • 8/17/2019 truongxuan2016-2

    16/18

    bt đng thc này có th thu gn thành

    (a − b)(b − c)(a2 − b2 + c2)abc

      0.

    Hin nhiên đúng theo gi thit ca  b nên ta có điu phi chng minh.  

    Bài 6.  Vi s nguyên dương   n, gi   Bn+1 là tp tt c các xâu nh phân đ dài 

    n, tc là 

    Bn+1 = {anan−1 . . . a0 | ai ∈ {0, 1}, ∀i = 0, 1, . . . , n} .

    Vi mi xâu   a =  anan−1 . . . a0  thuc   Bn+1 ta gi 

    s(a) = an + an−1 + · · · + a0   (mod 2)  là bit kim tra ca xâu   a

    v(a) = an2n + an−12n−1 + · · · + a1 · 2 + a0   là giá tr ca xâu   aGi   Bn+10   , B

    n+11   tương ng là tp hp tt c các xâu nh phân có đ dài   n + 1  có 

    bit kim tra tương ng là  0 và  1. Chng minh rng vi mi s nguyên dương  k, n,

    ta có đng thc 

    a∈Bn+10

    (v(a))k =

    a∈Bn+11

    (v(a))k.

    Li gii.  Vi  n là s nguyên dương, ta đtAn =

    v(a)|a ∈ B n+1

    = {0, 1, 2, . . . , 2n − 1} ,

    An0  =

    v(a)a ∈ Bn+10 , An1  = v(a) a ∈ Bn+11

    Bng cách xét bit đu tiên ca các xâu, d dàng kim tra đưc rng

    Bn+10   = {0an−1 . . . a0 |an−1 . . . a0 ∈ Bn0 } ∪ {1an−1 . . . a0 |an−1 . . . a0 ∈ Bn1 } ,Bn+11   = {1an−1 . . . a0 |an−1 . . . a0 ∈ Bn0 } ∪ {0an−1 . . . a0 |an−1 . . . a0 ∈ Bn1 } .

    Do đó

    An+10   = An0 ∪ {x + 2n |x ∈ An1 }   và An+11   = An1 ∪ {x + 2n |x ∈ An0 }

    Ta s chng minh x∈An0

    xk =x∈An1

    xk vi mi   k   = 1, 2, 3, . . . , n − 1 bng quy nptheo  n. (∗)

  • 8/17/2019 truongxuan2016-2

    17/18

    Tht vy,

    Vi  n = 1, ta cóB20  = {00, 11} , B21  = {10, 01}A10 = {0, 3} , A11 = {1, 2} .

    Rõ ràng  01 + 31 = 11 + 21 nên khng đnh đúng vi  n = 1.

    Gi s  (∗) đúng vi  n =  k ≥ 1. Ta xét  2 trưng hp:

    1. Nu  t = 1, 2, 3, . . . , k − 1, ta có:

    x∈Ak+10

    xt =x∈Ak0

    xt +x∈Ak1

    (x + 2k)t

    =x∈Ak0

    xt +x∈Ak1

      ti=0

    C itxt−i2ki

    =x∈Ak0

    xt +

    ti=0

    x∈Ak1

    xi

    C it2k(t−i)

    Tương t, ta cũng có

    x∈Ak+11

    xt =x∈Ak1

    xt +

    ti=0

    x∈Ak0

    xi

    C it2t−i

    .

    Theo gi thit quy np thì x∈Ak0

    xi =x∈Ak1

    xi

    vi  i = 1, 2, 3, . . . , k − 1 nên suy rax∈Ak+10

    xt =

    x∈Ak+11

    xt.

    2. Nu  t =  k: ta cũng bin đi như sau:

    x∈Ak+10

    xk =x∈Ak0

    xk +x∈Ak1

    (x + 2k)k

    =x∈Ak0

    xk +x∈Ak1

    xk +x∈Ak1

    k−1i=0

    C ikxk−i2ki

    =x∈Ak

    xk +

    k−1i=0

    x∈Ak1

    xi

    C ik2k(k−i)

  • 8/17/2019 truongxuan2016-2

    18/18

    Tương t, ta cũng có

    x∈A

    k+1

    1

    xk = x∈A

    k

    xk +

    k−1

    i=0

    x∈Ak

    0

    xi

    ik2

    k(k−i)

    .

    Đn đây, áp dng gi thit quy np, ta cũng cóx∈Ak+10

    xk =

    x∈Ak+11

    xk.

    Do đó,  (∗) cũng đúng vi  n =  k  + 1. Theo nguyên lý quy np thì khng đnh  (∗)đưc chng minh.

    Bài toán đưc gii quyt hoàn toàn.